Difference between revisions of "2005 AMC 12B Problems/Problem 4"

(Problem)
Line 1: Line 1:
 
== Problem ==
 
== Problem ==
 +
At the beginning of the school year, Lisa's goal was to earn an A on at least <math>80\%</math> of her <math>50</math> quizzes for the year.  She earned an A on <math>22</math> of the first <math>30</math> quizzes.  If she is to achieve her goal, on at most how many of the remaining quizzes can she earn a grade lower than an A?
 +
 +
<math>
 +
\mathrm{(A)}\ 1      \qquad
 +
\mathrm{(B)}\ 2      \qquad
 +
\mathrm{(C)}\ 3      \qquad
 +
\mathrm{(D)}\ 4      \qquad
 +
\mathrm{(E)}\ 5
 +
</math>
  
 
== Solution ==
 
== Solution ==

Revision as of 22:15, 17 April 2009

Problem

At the beginning of the school year, Lisa's goal was to earn an A on at least $80\%$ of her $50$ quizzes for the year. She earned an A on $22$ of the first $30$ quizzes. If she is to achieve her goal, on at most how many of the remaining quizzes can she earn a grade lower than an A?

$\mathrm{(A)}\ 1      \qquad \mathrm{(B)}\ 2      \qquad \mathrm{(C)}\ 3      \qquad \mathrm{(D)}\ 4      \qquad \mathrm{(E)}\ 5$

Solution

See also